Last visit was: 26 Apr 2024, 09:04 It is currently 26 Apr 2024, 09:04

Close
GMAT Club Daily Prep
Thank you for using the timer - this advanced tool can estimate your performance and suggest more practice questions. We have subscribed you to Daily Prep Questions via email.

Customized
for You

we will pick new questions that match your level based on your Timer History

Track
Your Progress

every week, we’ll send you an estimated GMAT score based on your performance

Practice
Pays

we will pick new questions that match your level based on your Timer History
Not interested in getting valuable practice questions and articles delivered to your email? No problem, unsubscribe here.
Close
Request Expert Reply
Confirm Cancel
SORT BY:
Kudos
Tags:
   
Show Tags
Hide Tags
Current Student
Joined: 13 Apr 2019
Posts: 237
Own Kudos [?]: 65 [0]
Given Kudos: 309
Location: India
GMAT 1: 710 Q49 V36
GPA: 3.85
Send PM
Tutor
Joined: 16 Oct 2010
Posts: 14830
Own Kudos [?]: 64934 [0]
Given Kudos: 427
Location: Pune, India
Send PM
Intern
Intern
Joined: 13 Aug 2013
Posts: 24
Own Kudos [?]: 15 [0]
Given Kudos: 90
Location: India
Concentration: Technology, General Management
Send PM
Intern
Intern
Joined: 13 Aug 2013
Posts: 24
Own Kudos [?]: 15 [0]
Given Kudos: 90
Location: India
Concentration: Technology, General Management
Send PM
Re: Veritas Prep CR Forum Expert - Karishma - Ask Me Anything about CR [#permalink]
VeritasKarishma wrote:
Rinng0 wrote:
VeritasKarishma

Hi Karishma,

Please help me to understand how option B is inline with Biologist's claim.

LSAT CR link https://gmatclub.com/forum/if-the-forest-continues-to-disappear-at-its-present-pace-the-koala-wi-61578.html#p29183
“If the forest continues to disappear at its present pace, the koala will approach extinction,” said the biologist.

“So all that is needed to save the koala is to stop deforestation,” said the politician.

Which one of the following statements is consistent with the biologist’s claim but not with the politician’s claim?

(A) Deforestation continues and the koala becomes extinct.
(B) Deforestation is stopped and the koala becomes extinct.
(C) Reforestation begins and the koala survives.
(D) Deforestation is slowed and the koala survives.
(E) Deforestation is slowed and the koala approaches extinction.

My understanding -
Event A(disappearance of forest) -> will cause -> Event B(extinction of Koala)
Not Event B(extinction of Koala) -> Not event A(disappearance of forest)
Thus, if Koala is not extinct then deforestation is stopped. It can be deduced from the biologist's statement.

But how to infer that option B(Deforestation is stopped and the koala becomes extinct.) is also a possibility from the same statement?


Check this:
https://gmatclub.com/forum/if-the-fores ... l#p2471153

Let me know if something is still unclear.


Thanks Karishma, nice explanation. It is now clear to me.
Intern
Intern
Joined: 18 Oct 2018
Posts: 9
Own Kudos [?]: 6 [0]
Given Kudos: 39
Send PM
Re: Veritas Prep CR Forum Expert - Karishma - Ask Me Anything about CR [#permalink]
Hi VeritasKarishma

Thank you for your devotion to the community here. It's great to see you contribute every single request.
Since I learn GMAT by my self, I found the hard part of CR is how to understand Multi-Negation Structure

Example 1:
Microbiologist: A lethal strain of salmonella recently showed up in a European country, causing an outbreak of illness that killed two people and infected twenty-seven others. Investigators blame the severity of the outbreak on the overuse of antibiotics, since the salmonella bacteria tested were shown to be drug-resistant. But this is unlikely because patients in the country where the outbreak occurred cannot obtain antibiotics to treat illness without a prescription , and the country’s doctors prescribe antibiotics less readily than do doctors in any other European country.

The question type for this CR is weaken the conclusion.

To simplify this complex Conclusion, I negate:
"cannot obtain antibiotics to treat illness without a prescription" = "Can obtain antibiotics to treat illness only with a prescription"
The first part of conclusion is " this is unlikely" so the right answer for weaken = Can obtain antibiotics to treat illness only with a prescription
I chose B through POE but when look at what I have negated, it's contradict. What wrong here
And how to have good multi-negation
https://gmatclub.com/forum/microbiologist-a-lethal-strain-of-salmonella-recently-showed-up-in-a-221739.html

Example 2:
Question 3 of 1 RC, GMAT also tests comprehension by state
3) The passage suggests that Twigg believes that rats could not have spread the Black Death unless which of the following were true?
Since this is super confusing, i also simplify by negation:
Unless + true = if wrong
Could not + if wrong = could + true
-> the question is now: The passage suggests that Twigg believes that rats could have spread the Black Death if which of the following were true?

The answer fit my process.

https://gmatclub.com/forum/the-black-death-a-severe-epidemic-that-ravaged-fourteenth-century-eur-153465.html

In short, could you share your idea about this technique, and how to reach minimum mistake.

Thank you so much and look forward your return :)
Manager
Manager
Joined: 23 Feb 2012
Posts: 119
Own Kudos [?]: 107 [0]
Given Kudos: 1356
Location: India
Concentration: Strategy, Technology
Send PM
Re: Veritas Prep CR Forum Expert - Karishma - Ask Me Anything about CR [#permalink]
Hi Karishma,


https://gmatclub.com/forum/excavation-o ... 94368.html
For the above question ,Please address my queries:
1)Why E is wrong ; I found it as taking strongest stand for the pattern that Camarnian official stayed only 1 place and mostly managed 4 simulaneously
2) For B : I had doubts , as B talks only about early carrer so how can we r sure that it will give information of 30 years career pattern
3) How to handle questions when all choices looks incorrect or partially correct? Any Technique ?

Thanks a ton!
Intern
Intern
Joined: 14 Jul 2019
Posts: 35
Own Kudos [?]: 15 [0]
Given Kudos: 72
Location: India
Schools: IMD '21
Send PM
Re: Veritas Prep CR Forum Expert - Karishma - Ask Me Anything about CR [#permalink]
Last year the rate of inflation was 1.2 percent, but for the current year it has been 4 percent. We can conclude that inflation is on an upward trend and the rate will be still higher next year.

Which of the following, if true, most seriously weakens the conclusion above?

(A) The inflation figures were computed on the basis of a representative sample of economic data rather than all of the available data.

(B) Last year a dip in oil prices brought inflation temporarily below its recent stable annual level of 4 percent.

(C) Increases in the pay of some workers are tied to the level of inflation, and at an inflation rate of 4 percent or above, these pay raises constitute a force causing further inflation.

(D) The 1.2 percent rate of inflation last year represented a 10-year low.

(E) Government intervention cannot affect the rate of inflation to any significant degree.

HI KArishma.

I will present my understanding and hypothesis and then would request for your inputs.

Premise: LY inflation was 1.2% and for current year it HAS BEEN 4%( has been shows the year is not over).
Conclusion: Inflation is on an upward trend & will be still higher next year.
WHY? Because from last year till now inflation has increased so it will keep increasing

We need to weaken the argument by attacking the premise.

What if the inflation last year itself wasn't the right figure. OR what if there was some other MACRO factor which drove inflation down, and which can be seen as an aberration.

OUt of the options above, i chose (A). WHY? Inflation last year was for a subset of data and not representative of a whole. So may be inflation was not 1.2% at all. SO how can we say its on an upward trend since then?

I was confused between A and B as both match somewhat to my pre thinking.

Pls help me clear my doubt.
Thanks
Intern
Intern
Joined: 14 Jul 2019
Posts: 35
Own Kudos [?]: 15 [0]
Given Kudos: 72
Location: India
Schools: IMD '21
Send PM
Re: Veritas Prep CR Forum Expert - Karishma - Ask Me Anything about CR [#permalink]
Hello Karishma,

PLs help me with the question below:

Mayor: In each of the past five years, the city has cut school funding and each time school officials complained that the cuts would force them to reduce expenditures for essential services. But each time, only expenditures for nonessential services were actually reduced. So school officials can implement further cuts without reducing any expenditures for essential services.

Which of the following, if true, most strongly supports the mayor’s conclusion?


(A) The city’s schools have always provided essential services as efficiently as they have provided nonessential services.

(B) Sufficient funds are currently available to allow the city’s schools to provide some nonessential services.

(C) Price estimates quoted to the city’s schools for the provision of nonessential services have not increased substantially since the most recent school funding cut.

(D) Few influential city administrators support the funding of costly nonessential services in the city’s schools.

(E) The city’s school officials rarely exaggerate the potential impact of threatened funding cuts.

I Chose D over B:

Why Reject B-- Schools have enough money to support SOME non essential services(NES), Thats means may be one OR may be all NES. Cant be assumed that they have enough money for all- non certain

Why Chose D-- FEW administrators support the funding of costly NES. Hence they have an alternate source of money. Thus they can continue cutting NES expense.
Director
Director
Joined: 03 Mar 2017
Posts: 586
Own Kudos [?]: 418 [0]
Given Kudos: 596
Location: India
Concentration: Operations, Technology
Send PM
Re: Veritas Prep CR Forum Expert - Karishma - Ask Me Anything about CR [#permalink]
VeritasKarishma

Hi

Please help me understand the below query of mine.

https://gmatclub.com/forum/although-par ... l#p2504985

Thanks
Current Student
Joined: 25 Apr 2020
Posts: 59
Own Kudos [?]: 76 [0]
Given Kudos: 138
Location: India
GMAT 1: 790 Q50 V51
GRE 1: Q167 V163
GPA: 3
Send PM
Re: Veritas Prep CR Forum Expert - Karishma - Ask Me Anything about CR [#permalink]
Hi.
I have a doubt in the following statement b/w sufficient and necessary.
"The children go to the park when the sun is shining.
My question is that why is "children going to the park" not the sufficient condition, as it means that the sun must be shining, while Sun shining doesn't mean children will always go to the park if the sun shines. In the CR bible, "Sun shining" is given as the sufficient condition, which I don't understand.
Thanks in advance.
Tutor
Joined: 16 Oct 2010
Posts: 14830
Own Kudos [?]: 64934 [0]
Given Kudos: 427
Location: Pune, India
Send PM
Re: Veritas Prep CR Forum Expert - Karishma - Ask Me Anything about CR [#permalink]
Expert Reply
warrior1991 wrote:
VeritasKarishma

Hi

Please help me understand the below query of mine.

https://gmatclub.com/forum/although-par ... l#p2504985

Thanks


There you go warrior1991:

https://gmatclub.com/forum/although-par ... l#p2506045
Tutor
Joined: 16 Oct 2010
Posts: 14830
Own Kudos [?]: 64934 [0]
Given Kudos: 427
Location: Pune, India
Send PM
Re: Veritas Prep CR Forum Expert - Karishma - Ask Me Anything about CR [#permalink]
Expert Reply
Rachit4126 wrote:
Last year the rate of inflation was 1.2 percent, but for the current year it has been 4 percent. We can conclude that inflation is on an upward trend and the rate will be still higher next year.

Which of the following, if true, most seriously weakens the conclusion above?

(A) The inflation figures were computed on the basis of a representative sample of economic data rather than all of the available data.

(B) Last year a dip in oil prices brought inflation temporarily below its recent stable annual level of 4 percent.

(C) Increases in the pay of some workers are tied to the level of inflation, and at an inflation rate of 4 percent or above, these pay raises constitute a force causing further inflation.

(D) The 1.2 percent rate of inflation last year represented a 10-year low.

(E) Government intervention cannot affect the rate of inflation to any significant degree.

HI KArishma.

I will present my understanding and hypothesis and then would request for your inputs.

Premise: LY inflation was 1.2% and for current year it HAS BEEN 4%( has been shows the year is not over).
Conclusion: Inflation is on an upward trend & will be still higher next year.
WHY? Because from last year till now inflation has increased so it will keep increasing

We need to weaken the argument by attacking the premise.

What if the inflation last year itself wasn't the right figure. OR what if there was some other MACRO factor which drove inflation down, and which can be seen as an aberration.

OUt of the options above, i chose (A). WHY? Inflation last year was for a subset of data and not representative of a whole. So may be inflation was not 1.2% at all. SO how can we say its on an upward trend since then?

I was confused between A and B as both match somewhat to my pre thinking.

Pls help me clear my doubt.
Thanks


Rachit4126
Discussed here: https://gmatclub.com/forum/last-year-th ... l#p2496542

Also note that if the same method of measurement has been used for all measurements, they are comparable. Until and unless we are told that the method is insufficient or incorrect, there is no reason to doubt it.
Tutor
Joined: 16 Oct 2010
Posts: 14830
Own Kudos [?]: 64934 [0]
Given Kudos: 427
Location: Pune, India
Send PM
Re: Veritas Prep CR Forum Expert - Karishma - Ask Me Anything about CR [#permalink]
Expert Reply
Rachit4126 wrote:
Hello Karishma,

PLs help me with the question below:

Mayor: In each of the past five years, the city has cut school funding and each time school officials complained that the cuts would force them to reduce expenditures for essential services. But each time, only expenditures for nonessential services were actually reduced. So school officials can implement further cuts without reducing any expenditures for essential services.

Which of the following, if true, most strongly supports the mayor’s conclusion?


(A) The city’s schools have always provided essential services as efficiently as they have provided nonessential services.

(B) Sufficient funds are currently available to allow the city’s schools to provide some nonessential services.

(C) Price estimates quoted to the city’s schools for the provision of nonessential services have not increased substantially since the most recent school funding cut.

(D) Few influential city administrators support the funding of costly nonessential services in the city’s schools.

(E) The city’s school officials rarely exaggerate the potential impact of threatened funding cuts.

I Chose D over B:

Why Reject B-- Schools have enough money to support SOME non essential services(NES), Thats means may be one OR may be all NES. Cant be assumed that they have enough money for all- non certain

Why Chose D-- FEW administrators support the funding of costly NES. Hence they have an alternate source of money. Thus they can continue cutting NES expense.


Discussed here Rachit4126
https://gmatclub.com/forum/mayor-in-eac ... l#p2506807
Senior Manager
Senior Manager
Joined: 28 Jan 2017
Posts: 365
Own Kudos [?]: 78 [0]
Given Kudos: 832
Send PM
Re: Veritas Prep CR Forum Expert - Karishma - Ask Me Anything about CR [#permalink]
https://gmatclub.com/forum/veritas-prep ... l#p2348776
VeritasKarishma wrote:
varotkorn wrote:
Dear VeritasKarishma

In Strengthening Question, when I negate an option and the negated version weakens the conclusion, will that option be considered a correct strengthener?

Thank you for all your replies :)


varotkorn

Not necessarily. The option could be neutral but negating it could weaken the conclusion. To look for a strengthener, negating does not help because strengtheners are not necessary for the conclusion to hold. Even if they don't hold, the conclusion could still hold. The only time we use negating is in assumption questions. That said, I prefer to look for a gap between premises and conclusion in assumption questions. Negating can get too tiresome so I use it in very few questions.

Dear VeritasKarishma,

I've found that the OE for this Strengthen question https://gmatclub.com/forum/the-spacing- ... 59671.html states that:
Quote:
One way to approach this [strengthen] question is to ask yourself, "If this option were FALSE, would the hypothesis be LESS likely to be true?

(IMO, FALSE = negate & LESS likely to be true = WEAKEN)
In a way, OE implies that if the NEGATED version WEAKENS the conclusion, that option will be considered a correct strengthener.
This is similar to what I had in mind when I asked Ma'am before. The highlighted portions of OE and my previous question posted topmost are of quite similar ideas.

I'm very confused here.

Please help :please:

Side note: some expert also uses the negation in Strengthen Question (https://www.manhattanprep.com/gmat/blog ... reasoning/). Two quotes below are from the aforementioned link:
Quote:
So let’s say the opposite of each statement were true

Quote:
This reasoning also works in Weakening or Assumption type questions. If my job is to weaken the conclusion, then I want to find evidence that would strengthen the conclusion and then prove that this is not true.

Originally posted by kornn on 30 Apr 2020, 03:31.
Last edited by kornn on 29 May 2020, 08:02, edited 2 times in total.
Intern
Intern
Joined: 15 Jun 2015
Posts: 14
Own Kudos [?]: 12 [0]
Given Kudos: 46
Send PM
Re: Veritas Prep CR Forum Expert - Karishma - Ask Me Anything about CR [#permalink]
VeritasKarishma wrote:
Ask me Anything about GMAT Critical Reasoning


Hello and welcome to my CR Expert Topic. Let me know if you have any CR questions I can help with - strategy, specific ones, etc.

You may know me from forum discussion - I am one of the resident Veritas Prep Experts on GMAT Club. I have over 8,000 posts and 22,000 Kudos. I hope you find my replies and explanations helpful. If you have any specific questions about Critical Reasoning in General such as strategy/approach, feel free to post them here and I will be happy to address them. I intend to have this thread be as a "Everything You Need to Know about CR" type of thread.

If you have questions about a specific CR question and why something does not seem right, please post a link here for me with your specific question/concern. Please do not double-post - that's not polite and against the GMAT Club rules.

Thank you all - good luck on the GMAT and look forward to seeing you in the CR forum!
Karishma.


@Karishma..Hi Karishma..need your help with an issue that I am facing in CR...most of the times in the actual test, The way I interpret the CR Q is quite different from how I would interpret the Q under normal circumstances (I interpret it correctly most of the times). I have a more than 80% accuracy in CR generally, but during the exam, due to the misinterpreting the Q, my accuracy takes a hit. Is this something people generally face or is it unique to me?. Also, how best to tackle it?. I am 1 month away from my GMAT
Manager
Manager
Joined: 30 Jun 2019
Posts: 52
Own Kudos [?]: 59 [0]
Given Kudos: 74
Location: India
Concentration: Operations, Finance
WE:Engineering (Manufacturing)
Send PM
Re: Veritas Prep CR Forum Expert - Karishma - Ask Me Anything about CR [#permalink]
https://www.veritasprep.com/blog/2016/02/circular-reasoning-gmat-cr/

Above is a link to one of your articles . I am specifically refering to the below question.

Anatomical bilateral symmetry is a common trait. It follows, therefore, that it confers survival advantages on organisms. After all, if bilateral symmetry did not confer such advantages, it would not be common.

You have mentioned that the reasoning in the above argument is flawed. But i disagree.

I think it is a conditional statement. Let me rearrange the argument.

Premise : After all, if bilateral symmetry did not confer such advantages, it would not be common. ( If not A , then not B. This implies that " if B , then A' statement must also be true)
Premise : Anatomical bilateral symmetry is a common trait. ( B happens )
Conclusion : It follows, therefore, that it confers survival advantages on organisms. ( Therefore A must happens)

I would like to understand what makes my understanding wrong ?
Intern
Intern
Joined: 12 Feb 2020
Posts: 33
Own Kudos [?]: 58 [0]
Given Kudos: 138
Location: India
Send PM
Re: Veritas Prep CR Forum Expert - Karishma - Ask Me Anything about CR [#permalink]
Can you help on this?- Difference between option A and D
https://gmatclub.com/forum/lawyer-this- ... l#p2516743
Manager
Manager
Joined: 23 Dec 2011
Posts: 66
Own Kudos [?]: 82 [0]
Given Kudos: 131
Send PM
Re: Veritas Prep CR Forum Expert - Karishma - Ask Me Anything about CR [#permalink]
Manager
Manager
Joined: 23 Dec 2011
Posts: 66
Own Kudos [?]: 82 [0]
Given Kudos: 131
Send PM
Re: Veritas Prep CR Forum Expert - Karishma - Ask Me Anything about CR [#permalink]
Gmat410 wrote:
Can you help on this?- Difference between option A and D
https://gmatclub.com/forum/lawyer-this- ... l#p2516743


I am not an expert, but here is my take on this question -

Prompt says - Witness could not recognize both the parties -> exclude the testimony.

A) witness claims to recognize both parties -> include testimony. This is a mistaken negation and does not have to be true.

D) Include testimony (Sufficient Condition) -> Only if witness can recognize both parties (Necessary Condition). this is contrapositive and is true.

Hence, D is the correct assumption.
Current Student
Joined: 13 Apr 2019
Posts: 237
Own Kudos [?]: 65 [0]
Given Kudos: 309
Location: India
GMAT 1: 710 Q49 V36
GPA: 3.85
Send PM
Re: Veritas Prep CR Forum Expert - Karishma - Ask Me Anything about CR [#permalink]
GMAT Club Bot
Re: Veritas Prep CR Forum Expert - Karishma - Ask Me Anything about CR [#permalink]
   1  ...  23   24   25   26   27   28   29  ...  34   
Moderators:
GMAT Club Verbal Expert
6921 posts
GMAT Club Verbal Expert
238 posts
CR Forum Moderator
832 posts

Powered by phpBB © phpBB Group | Emoji artwork provided by EmojiOne